Para esta demonstração vamos usar o jogo $\mathsf G_1(\mathcal O^*, \mathcal O)$ que é dual ao jogo de Menger.\\ Em primeiro lugar, as coberturas jogadas continuam sendo enumeráveis, pois temos a combinação finita de enumeráveis elementos, o que é enumerável.\\ Definimos $\sigma$ da mesma maneira do exercício anterior e $D_{\xi}=\{s\in ^{< \omega}\omega|K_{\xi}\subset A_s\}$. Note que, dada uma cobertura qualquer para $X$, em particular ela recobre $K_\xi$, mas $K_\xi$ é compacto, então existe $(A_k)_{k